Anda di halaman 1dari 53

Issue Estoppel 1.

The binding effect of a judgment as to matters actually litigated and


determined in one action on later controversies between the parties involving a different
claim from that on which the original judgment was based. 2. A doctrine barring a party
from relitigating an issue determined against that party in an earlier action, even if the
second action differs significantly from the first one.

Substantive law - The part of the law that creates, defines, and regulates the rights,
duties, and powers of parties. Cf. PROCEDURAL LAW. "So far as the administration of
justice is concerned with the application of remedies to violated rights, we may say that
the substantive law defines the remedy and the right, while the law of procedure defines
the modes and conditions of the application of the one to the other."
Procedural law - The rules that prescribe the steps for having a right or duty judicially
enforced, as opposed to the law that defines the specific rights or duties themselves. --
Also termed adjective law. Cf. SUBSTANTIVE LAW

Forum non conveniens [Latin "an unsuitable court"] Civil procedure. The doctrine that
an appropriate forum -- even though competent under the law -- may divest itself of
jurisdiction if, for the convenience of the litigants and the witnesses, it appears that the
action should proceed in another forum in which the action might also have been properly
brought in the first place. -- "Forum non conveniens allows a court to exercise its
discretion to avoid the oppression or vexation that might result from automatically
honoring plaintiff's forum choice. However, dismissal on the basis of forum non
conveniens also requires that there be an alternative forum in which the suit can be
prosecuted. It must appear that jurisdiction over all parties can be secured and that
complete relief can be obtained in the supposedly more convenient court. Further, in at
least some states, it has been held that the doctrine cannot be successfully invoked when
the plaintiff is resident of the forum state since, effectively, one of the functions of the
state courts is to provide a tribunal in which their residents can obtain an adjudication of
their grievances. But in most instances a balancing of the convenience to all the parties
will be considered and no one factor will preclude a forum non coveniens dismissal, as
long as another forum is available." Jack H. Friedenthal et al., Civil Procedure §§ 2.17, at
87-88 (2d ed. 1993).

In personam [Latin "against a person"] 1. Involving or determining the personal rights


and obligations of the parties. 2. (Of a legal action) brought against a person rather than
property. Cf. IN REM. in personam, adv. "An action is said to be in personam when its
object is to determine the rights and interests of the parties themselves in the subject-
matter of the action, however the action may arise, and the effect of a judgment in such
an action is merely to bind the parties to it. A normal action brought by one person
against another for breach of contract is a common example of an action in personam."

Lex fori - The law of the forum; the law of the jurisdiction where the case is pending <the
lex fori governs whether the death penalty is a possible punishment for a first-degree-
murder conviction>. -- Also termed lex ordinandi. Cf. LEX LOCI (1).
lex loci - [Latin] 1. The law of the place; local law. Cf. LEX FORI. 2. LEX LOCI
CONTRACTUS.

Lex loci actus - The law of the place where an act is done or a transaction is completed.
-- Often shortened to lex actus.

Lex causae - The legal system that governs a dispute.

Lex loci contractus - The law of the place where a contract is executed or to be
performed. • Lex loci contractus is often the proper law by which to decide contractual
disputes. -- Often shortened to lex loci; lex contractus. "The lex loci contractus controls
the nature, construction, and validity of the contract; and on this broad foundation the law
of contracts, founded on necessity and commercial convenience, is said to have been
originally established. If the rule were otherwise, the citizens of one country could not
safely contract, or carry on commerce, in the territories of another."

Lex loci delicti - The law of the place where the tort or other wrong was committed. --
Often shortened to lex delicti. -- Also termed lex loci delictus; lex loci delicti commissi;
place-of-wrong rule; place-of-wrong law. Cf. LOCUS DELICTI.

Lex loci solutionis - Latin "law of the place of solution", The law of the place where a
contract is to be performed (esp. by payment). -- Often shortened to lex solutionis.

Conflict of laws; it is the court of NB looking at its own conflict of laws rules, whether
its dispute should be decided based on the substantive law of Ontario or not.

“Substantive Reference” – is the NB looking at its own conflicts rules to determine if it


is going to use the substantive laws of another place or that of NB.

“Domestic Rule of Decision”: indicates the rule that would be applied by the court if
there were no relevant foreign facts of any kind –as if the matter had arisen within
exclusively local facts.

INTRODUCTION – pp. 1-17

1. Questions Central to the conflict of Laws


- where there are relevant facts that have occurred outside the province, the rules
and principles of Conflict of Laws are used to reach a solution
- Conflict of Laws is concerned with cases where there are geographically complex
facts
- Three problems
o When should the court take jurisdiction in cases with geographically
complex facts?
o When should the court recognize/enforce a foreign judgement?
o When should the court use foreign law to help decide the issue?
- Generally, Conflicts deals with private law
- Some statute, some common law rules
- “conflict of laws” vs. private international law
o Common law vs. civil tradition
o Private international law is a better descriptive term
2. Note on the Materials
3. The Purpose of the Materials
- two goals
o understanding conventional organization of the law
o examining those rules to see if they are useful predictors of the court
decisions that apply them
- traditional method of analysis – English
- modern approach incorporates new features and concerns
o fairness to individual parties
o predictively useful rules
o faithfulness to constitutional principles in a federal state (i.e.
interprovincial context in Canada)
- dual focus – result in the individual case & development of set of rules over time
- Four main areas for conflicts problems
o Substantive law of tort & contract
o Procedural Issues of jurisdiction
o Recognition & Enforcement of foreign judgements

4. Note on Uniform Laws


- international uniform law has been developed in some areas
- this resolves conflicts problems before they arise
- despite this advance, common law rules will remain important

5. What is a ‘Conflicts’ problem?


- conflicts case = any case with geographically complex facts
- must allege geographically complex facts in pleadings, plead foreign law
- once it is determined that there are geographically complex facts, you then move
to the analysis of conflicts issues
- fundamental difference between establishing foreign law and establishing the law
of the forum
o call expert for evidence of foreign law
o judicial notice of forum law – no evidence
- contents of the foreign law: matter of fact, expert evidence
- relevance of foreign law: decided by using the forum’s jurisdiction laws
o plaintiff argues that foreign law is relevant, how does the court respond?
o Choice of law rules
Part 1 – Choice of Law
Chapter 1 – Process and Analysis – pp. 18-24
1. Introduction
- jurisdiction; recognition & enforcement; choice of law
- choice of law – most important and most difficult of the questions
2. Choice of Law Process
- issue raised by choice of law questions: if a Canadian court does exercise
jurisdiction, and it is alleged that foreign law is relevant, what rule of decision
does it apply?
o Forum law vs. foreign law
o Can involve multiple jurisdictions
- ‘domestic rule of decision’
o Rule that a court would apply in case with geographically complex
facts
- In some cases a more just decision will be reached by applying a different
jurisdiction’s ‘domestic rule of decision’
- Choice of law process = justifying the decision to apply someone else’s
‘domestic rule of decision’
3. Structure of Conflicts Analysis
- pleadings/preliminary matters
o existence of foreign facts – alleged and found by the court as fact
o content of foreign law – alleged and proved properly
- if these conditions are met, court proceeds to determining to apply (or not
apply) foreign law
o Is the foreign law relevant to the issues before the court?
- How to analyze the relevance of foreign law to the case before the court
o Characterization
 What type of case is it?
o Choice of Law Rule
 What is the rule for that category of cases
o E.g. contracts = proper law of the contract
o Determining ‘proper law’ relies on local law – question of law, not
fact
- Focus of the dispute = determine which of the competing laws is the proper
law
- Tests for proper law of the contract
o Implied Intention Test
 What law would the parties have chosen?
 Look at connecting factors to the jurisdictions
 use connecting factors to determine governing law for the
dispute
 forum law vs. foreign law
- If foreign law is chosen, it will displace the domestic rule of the forum in
favour of the foreign country’s domestic rule
- Characterization is very important because it determines what choice of law
rule will be applied
- Goal of the process: predictability and uniformity
o Precisely defined rules help predict the outcome in future cases
o Uniformity – result shouldn’t depend on the place where the action is
brought
Part 1 – Choice of Law
Chapter 2 – Choice of Law in Contract – pp. 25-80
1. A Note on Historical Development
- modern development – 19th century to today
- based on the idea of legal formalism (law as a science), that has been largely
rejected in all other areas
- earliest formulation of contract choice of law rule
o the place where the contract was made – l.l.c.
o governing law covers validity, enforceability, and entitlement to
damages re contract
o focus on a single connecting factor
o should lead to uniformity
o this rule has declined in importance, but still one of the connecting
factors that is looked at
o place where contract is made = offer & acceptance
 where the last act necessary to make the contract occurs
 when the offeror receives notice of acceptance
• postal exception
- this rule was replaced by ‘will theory’ of contracts
o intention of the parties
- potential choice of law rules in contracts
o law of the parties domicile
o place of performance
o law implicitly chosen by parties
o law explicitly chosen by parties
o forum law
o law with the ‘closest and most real connection’ to the contract
- ‘closest and most real connection’
o Most modern formuliation of the choice of law rule

Etler v. Kertesz – OCA (1960) – p. 29


P lent money to D in Austria in US$. D was to repay the loan by having
sister in Switzerland give P the amount in US$. Loan not paid, parties
went to Ont. And P sues on the loan. The transaction would be void under
Austrian law.

I: Is proper law of contract Austria (place of contract) or Switzerland (place


of performance)?

H: For D: proper law is Austrian.

The proper law of the contract is whatever the parties intended or could be
presumed to intend.
In seeking to ascertain the intention of the parties as to the proper
law of the contract, in the absence of expressed intention it should be
determined as the one with which the transaction has its closest and
most real connection.
- both parties present in Austria
- contract entered into in Austria
- Performed substantial part of contract there

The Assunzione – England CA (1954) – p. 35

“A caricature of the traditional approach”

P owned cargo on board the Assunzione which they had chartered. It sunk
due to negligence of D, who owned the ship.

I: Which law should be applied to the contract? French law (P’s nationality)
or Italian law (D’s nationality)?

H: court does a mechanical counting of connections to find the proper law;


does not discuss the contracts issue being contested. This case
represents a move away from the “intention” approach; it relies on an
“objective” approach.

Amin Rasheed Shsipping Corporation v. Kuwait Insurance Co. – House of Lords


(1984) – p. 40

• Good illustration of the traditional c.o.l. process. Diplock says:


- Intent: First see what intentions of the parties are: freedom of
contract, so their intentions should prevail
- Real & Substantial Connection: If there is nothing to indicate
what the intentions of the parties were, use the real and substantial
connection test: to what transaction is there the most real and
substantial connection?
• No renvoi in contract law
• There is no “international” law of contract (not sure if this case
stands for that but it discusses it)

- Summary of the traditional method for solving choice of law problems in


contract
o ‘proper law of the contract’
 System which the parties intended, express or implied
 If not, ‘closest and most real connection’
o Formation is governed by proper law
o Capacity is governed by closest connection or domicile/residence
o Formal validity is governed by place where contract is made or
proper law
o Material/essential validity is governed by proper law
o Interpretation is governed by proper law
o Right and obligations of parties are governed by proper law
o Mode of performance is governed by place of performance
o Discharge of a contract is governed by proper law

2. The Effect of an Express Choice of Law Clause


- standard for contracts to contain a ‘governing law clause’

Vita Food Products Inc v. Unus Shipping Col Ltd. – Privy Council (1939)
Leading case on exclusive c.o.l. clause in contract

Herring shipped from Nfld. to NY arrived damaged due to negligent


shipper. Buyer wants to sue. Hague rules would limit amt. of damages.
Nfld. statute says all bills of lading must have a clause saying Nfld. law
applies, and P argues that bills of lading were illegal under Nfld. law and
so not caught by Hague rules. D says even though bills of lading didn’t
comply w. Nfld. statute they said law of England governs, and law of
England is party to Hague rules so they apply.

H: Court gives effect to c.o.l. clause saying law of England applies.

Give effect to a choice of law clause if it’s bona fide legal and doesn’t
offend public policy

- all that a choice of law clause can do is to indicate the background legal
system against which the contract was drafted/made
o can’t expand or limit the parties freedom to make deals on their
own terms
- choice of law clause would not avoid the following problems
o fundamental breach
o enforcement of penalties
o fiduciary obligations (agent-principal)
o mistake/frustration
o assignability
- these potential problems are requirements along the line of ‘if bona fide
legal and doesn’t offend public policy’
o some restrictions on ability to choose ‘governing law’

3. Some Further Problems of Choice of Law in Contracts


- limits on the parties implied or express choice of law
- following cases are not good examples of how to deal with these problems
Ross v. McMullen – Alberta SC (1971) – p. 59

Relation Between choice of law issues and Rules of the forum

Two Ont. real estate agents go to Calgary to develop property w.


agreement to split commission 70/30. P leaves AB after a few months.
Sale goes through, there’s commission, and D doesn’t split it w/ P, who
brings suit for it in AB. Both are bound by another contract saying
disputes are governed by law of Ont. AB law says you have to be a
licensed R/E agent in order to sue for commission.

I: Must decide whether to give effect to the part of the contract that
says it’s governed by the law of Ont. The land and negotiations and
sale and performance were in AB, but contract was made in Ont.

H: AB law applies:
 It is the lex fori and is a matter of public policy.
 First court says it doesn’t think the parties actually meant for the
law of Ontario to govern (not a bona fide choice), but even if they
did mean it, they can’t do that b/c it goes against a mandatory law.

Licensing real estate agents: there is a policy reason for the rule that if
you’re not licensed, you can’t sue for a commission. Thus you don’t
want people to be able to say their real estate agency contracts are
governed by the law of some other country w. no rules about licensing,
thereby entirely defeating the domestic law requiring real estate agents
to be licensed.

Decide whether a law is mandatory or facilitative. If merely


facilitative, then it is possible to contract out of it (e.g. SOG Act –
applies unless contracted out of).
 If you can’t do something expressly, you probably can’t do it
indirectly by a choice of law clause

A contract illegal under the lex fori is unenforceable there. To decide


whether a given law of the forum can be contracted out of, determine if
it’s mandatory or facilitative.

4. Review and Further Examination of Typical Cases

- outline of the choice of law process


o characterization determines if contracts choice of law rule applies
o contracts cases governed by proper law of the contract
 various potential choices
 ‘closest and most real connection’
o Express choice of law will be given effect if it is ‘bona fide and
legal and not contrary to public policy’
o If no express choice, ‘closest and most real connection’
 Consider all aspects of the contract
o Governed = all aspects of the contract are decided with reference
to the proper law
o Contracts must be localized in one system of law; no ‘unlocalized
or international’ system of law

Bondholders v. Manville – Sask. CA (1933) – p. 67

Unenforcability under a foreign law; relation between


unenforcability and choice of law issues

Facts: D was Sask. woman who signed a promissory note to buy


land in Fla. Later when she was sued on the note by a bondholder,
she claimed that she wasn’t bound by it b/c women can’t make
promissory notes under Fla. law.

I: What’s the purpose of Fla.’s incapacitation of women, and how


should that be applied to this case?

Decision: Promissory note not valid.


The court looks at policy and purpose behind the Fla. statute: is it
aimed at married women? All women?
The default position is freedom of contract. The Fla. statute is an
exception to that, but it may not apply.
The court finds here that it does apply: at least in the case of
mercantile contracts not prohibited by the lex fori, the weight of
authority is to the effect that the capacity or incapacity of a
person to contract must be determined by the law of the
country where the contract arises.

There is a debate about whether or not we can abandon


Assunzione and look at the policy and purpose of a statute.

o same choice of law process with a different rule

Charron v. Montreal Trust – Ont. CA (1958) – p. 69

Facts: Charron died and his former wife sues D, the executor of his
estate, for back payments of alimony under a separation agreement
made in Ontario. Quebec law doesn’t recognize separation
agreements as valid contracts. D claims law of domicile should
apply (husband’s permanent domicile was always Quebec) and P
says law of place of contract should apply.

Decision: Expert evidence of Quebec law says the contract would


be unenforceable. Court decides that the law governing the
contract is the proper law and that is Ont. law: The parties were
married there and lived there a long time

Imperial Life Assurance v. Colmenares – SCC (1967) – p. 72

A suit for a cash surrender value in life insurance policy. D enters


into contract w. P while in Cuba. Cuban revolution comes along and
freezes the assets of all the banks and institutions. Colmenares can
go back to Cuba and get his $8000 claim, but if he did that as soon
as he took it out the Cuban gov’t will seize it. He wants to get his
money from Toronto, P says to go back Cuba and make his claim
there.

Court uses real and substantial connections test: consider the


contract as a whole in light of all the circumstances which surround
it and apply the law with which it has the closest and most
substantial connection.

What court uses in its decision:


i) form and language
ii) Head office is in Ont.
iii) Payment in US dollars mostly
iv) Place of making

Court manipulates the rules a bit.


Payment could be made in Ontario.
This is really a claim about an unallocated risk in a contract. So
what difference does it make what the proper law of the contract is?
You could have the proper law be that of Cuba, with the parties
allocating the risk one way, or the proper law be Ontario’s w. the
parties allocating the risk the other way.

o real issue: which party should bear the risk of loss for this event?
o Requires more than just choosing a proper law

Rail Brothers v. Compania Naviera Sota… – Eng. CA (1920) – p. 78

English P chartered ship to carry cargo from India to Spain…


Just because it’s an English contract doesn’t prevent the court from
looking at a statute in force in the place of performance. [confirms
the Dicey rule?] This is similar to Southin J. in Gillespie.

This case doesn’t fit into traditional jurisdiction selection rule, but
has been used to stand for the position that issues of mode of
performance are governed by place of performance.
Part I – Choice of Law
Chapter 3 – Choice of Law in Torts – pp. 81-142
1. Introduction
- in contracts, rules across jurisdictions are nearly the same
o have to be, because of prevelance of international commercial contracts
- in tort, the differences among jurisdictions are more significant
o i.e. fault-based vs. no-fault
- theoretical issues that arise in torts
o criminal element
 historically, criminal element to torts
• intentional torts
o Fault vs. no-fault
 Negligence
 Vicarious liability = no-fault (strict liability)
 Defectively manufactured goods – strict liability in some
jurisdictions
o Social policy
 Tort law is influenced by social policy and is used to achieve social
objectives

2. Summary of the Development of Modern Tort Choice of Law Rules


- major judgement in 1994 – SCC
- four potential choice of law rules in tort
o place where the tort was committed – l.l.d.
 generally easy to ascertain, but can be difficult
 place of commission is generally held to be the place where injury
occurs
o place where the act giving rise to injury occurred – l.l.a.
 most cases, l.l.d. and l.l.a are the same
 e.g. not the same in case of negligent manufacture
o proper law of the tort
 ‘closest and most real connection’
 Importing the contracts rule
o Law of the forum – lex fori
- Prior to SCC restatement in 1994, Canada employed a ‘double-barrelled rule’
o Two conditions
 Wrong would have been actionable if committed in the forum (lex
fori)
 Act must not have been justifiable in the place where it was done
(l.l.a.)
o Considerable criticism of this rule
Chaplin v. Boys – House of Lords (1971) – p. 86

This doesn’t represent the law of Canada, and never has. It shows
England’s departure from Phillips v. Eyre, and shows the pros and cons of
each rule.

Facts: Two Englishmen get into an automobile accident in Malta. Maltese


law wouldn’t allow recovery for pain and suffering, so Chaplin sues in
England.

Wilberforce:
 Asks if Philips v. Eyre rule is satisfactory and canvases other options:
 Place of tort: Two disadvantages: first is the difficulty of
domestic courts trying to apply foreign law; second the place
where the wrong occurred can be fortuitous: place of the tort
can be a fluke. Also there’s the problem of having to decide
where the tort occurred.
 Contact/Interest Principle (Proper Law of the Tort): Looks at the
US where they look at all the factors and the policies that
underlie them. Wilberforce’s criticism of this is that there is too
much uncertainty: the search for relevant contacts and of
weighing them qualitatively against each other leads to
uncertainty and dissent. It’s also difficult to agree on the issue of
the underlying policy.

o Wilberforce says that the Phillips rule applies unless there is a reason
to ignore the foreign law. Basically within the double barreled
approach, you can’t derogate from the law of the forum, but you can
derogate from the l.l.d.: in this case, the Maltese government probably
doesn’t care whether or not their law prohibits two foreigners from not
being liable for, or able to recover, compensatory damages. Because
the two parties were British and lived in Britain etc., the Maltese
limitation could be ignored.

o traditional Phillips v. Eyre rule is forum centred


 forum rules are almost entirely determinative
 can lead to forum-shopping
o UK now has a statutory test for choice of law in tort
 l.l.d. subject to exceptions

McLean v. Pettigrew – SCC (1945) – p. 97

Canadian approach until Tolofson (1945-1994)


Facts: P is passenger in car. D is driver. They get into a single-car
accident in Ontario, and P brings suit in Quebec. S. 42 of Ontario’s
Highway Traffic Act says drivers aren’t liable to gratuitous passengers; P
therefore wants the law of Quebec to apply, b/c it doesn’t have any such
law. P wants to invoke contractual liability of D, but court doesn’t buy that.

Decision:
Court quotes Dicey rule stated in Philips:
An act done in a foreign country is a tort, and actionable as such in
England, if it is both
i) Non-justifiable according to the law of the foreign country where is
was done;
ii) Actionable as a tort, according to English law, i.e. an act which, if
done in England, would be a tort.

In Ontario, this wasn’t actionable, but the court gets around this by finding
that D’s actions were contrary to a provision stating that drivers must use
reasonable consideration for other persons using the highway: therefore
non-justifiable.

Chaplin v. Boys rejected the “gloss” of non-justifiability, i.e. it held that the
action must have civil liability in both places, but in Canada we bought
Phillips.

o SCC can take judicial notice of the laws of all provinces

O’Connor v. Wray – SCC (1930) – p. 101

Facts: D is Quebecois owner of car who lent it to the driver, who


negligently struck two women walking on the road in Ont., killing one and
injuring the other. P is husband of the killed woman, he relies on
provisions of Ont. Highway Traffic Act stating that the owner of a vehicle is
responsible for any violation of the Act; there is no such provision in
Quebec.

Held: For D.
In this case, civil liability would arise for this in Ontario (satisfies first part
of old test). This injury isn’t actionable in Quebec though (fails on second
part of Philips test)

However, court ignores choice of law rules and looks at the scope of
the law: Ont. can’t impose strict liability on people who aren’t in or
from Ontario. If they did create a statute that extends the scope of the
law to all people, no matter where they’re from, who have their
automobiles in Ontario, the statute might not be constitutionally valid.
o court could simply have decided that statute was not intended to apply to
Quebec residents
 defendant never became subject to the act
o trying to determine which of the innocent parties should suffer the loss +
geographically complex facts
 very difficult
o no-fault compensation re motor vehicle accidents caused a shift in the law
 no causes of action under no-fault
 applying traditional rules, Quebec residents (no-fault) could be
found liable in Ont. for Quebec accidents with Ont. residents
 provinces respond with legislation, but courts say people can still
bring action in their own province for accidents in Quebec

Grimes v. Cloutier – Ont. CA (1989) – p. 109

Facts : P is Ont. Resident injured in MVA in PQ by car driven by D.

Decision:
Distinguishes MacLean. We only want MacLean v. Pettigrew reading to
apply when the parties are from the same province.
Looks at many factors, including what’s fair between the parties.
Applying the Ontario legislation instead of the Quebec legislation
would be “officious intermeddling with the legal concerns of a sister
province.”

o apply Phillips unless it leads to an unjust or unreasonable result


o Phillips ignores important factors
 Residence of parties
 Insurance background
o These factors become relevant under the Grimes approach

Tolofson v. Jensen; Lucas v. Gagnon – SCC (1994) – p. 116

Place of the tort rule applies. Some courts have tried to get around this
rule since, but the SCC doesn’t leave much room for that.

Facts: P is injured son of driver, who sues driver and other driver for an
accident that occurred in Sask. Sask. had a law at the time that required
gross negligence to be proved in order for a gratuitous passenger to
recover. Sask. also required actions to be brought w/in 12 months; here
the boy brought the case 6 years later, at age 18.
Lower court in Gagnon basically applied the Grimes v. Cloutier test to find
that Quebec driver can get the benefit of Quebec’s no-fault scheme.

La Forest wipes the slate clean:


 Begin thinking about things on the basic level: nation to nation. The
principle on which this is based is territoriality – sovereign states get to
lay down the law in their territory.
 States generally try to accommodate judgments and rules of other
states: comity. States aren’t required to be comitous, but usually are.
 “From the general principle that a state has exclusive jurisdiction w/in
its own territories and that other states must under principles of comity
respect the exercise of its jurisdiction w/in its own territory, it seems
axiomatic to me that as a general rule the law to be applied in
torts is the law of the place where the activity occurred, i.e. lex loci
delicti.”
 One exception to this is when a wrong comes from an
interprovincial or transnational activity. In such cases it is uncertain
what the choice of law rule is.
 The challenge then becomes to figure out which torts fall into the
category with the clear rule set out by La Forest here and which are
interprovincial or transnational.
 The other exception to the clear rule here is if the l.l.d. would give
rise to injustice; in those cases the court can retain discretion to apply
Canadian law to deal w. those circumstances. La Forest says these
cases would be very rare.
 Court refers to the Constitution and notes that although this choice of
law rule isn’t dictated by it, and isn’t the only one that fits with it, this
rule does have the virtue of fitting with it.
 VB thinks this is a terrible decision: the purpose of not allowing
gratuitous passengers to sue drivers is to prevent fraud – guest
passengers and friend drivers could collude to prove negligence in a
no-fault system even where there was no negligence. In this case, the
BC govt. doesn’t want to heighten the bar so doesn’t have any
legislation protecting their govt. insurance co. (ICBC) from being
defrauded. Sask. doesn’t really care about limiting claims against
ICBC, so there is no real reason to apply Sask.’s limitation as between
a BC resident driver, and his son, a gratuitous passenger in his BC
registered automobile.

Ratio: Within Canada, as long as it is possible to determine the place


of the tort, the law of the place of the tort applies.

Courts have found ways around Tolofson when not happy with the result it
would give

- Tolofson is a rejection of the Grimes approach


Wong v. Lee – Ont. CA (2002) – p. 136

Exceptions to l.l.d. rule should be v. narrow

Facts: Passenger sues driver for accident that occurred in New York. Both
were residents of Ontario, and suit was brought in Ontario.

Decision:
 Place of tort should still apply.
 Just because the foreign law differs somewhat from the domestic law
doesn’t mean there is an injustice. The exception to Tolofson should
be very narrow.

Shuts down exceptions to Tolofson unless the foreign law amounts


to an injustice. Injustice is interpreted narrowly.

Insurance Context
o car insurance policies and Insurance Act offer a different approach to
‘inter-provincial torts’
o partial legislative reversal of Phillips; doesn’t require actionability by the
forum
Part 1 – Choice Of Law
Chapter 4 – Escape Devices – pp. 143-182
1. Introduction
- whole structure of choice of law process depends on characterization
- courts will sometimes manipulate characterization to achieve the result they want
- four principal methods to get to the result that the court wants
o substance/procedure distinction
o renvoi
o choosing between competing characterizations
o public policy doctrine

2. The Substance/Procedure Distinction


- Canadian court will never apply foreign procedural law
- Only foreign substantive law is applied
o Foreign substantive does not displace the forum’s procedural rules
- Anytime foreign law is found to apply, the substance/procedure distinction must
be made
o then characterized on the basis of contracts/torts, etc. if question of
substantive law
o or, law of the forum applies if question of procedural law
- procedural differences shouldn’t influence the eventual outcome
- categorizing laws as procedural negates the actual choice of law exercise
- example of procedural matters
o costs
o who may be a party to a legal action
o remedies – in some instances
 vested rights principle
o limitation periods
 historically, procedural
 civil law, substantive
 move towards substantive

TD Bank v. Martin Estate – Sask. QB(1985) – p. 146

D trying to escape liability by trying to characterize parts of forum law as


procedural – functional approach

P suing on mortgage made in BC for land in BC, where D resided. Now D


lives in Sask., and Sask. Land Contracts Act says no action can be
commenced w/o leave of court. D says it’s a procedural law, and if P sues
in Sask. court it has to use Sask. procedure.

 Another Sask. court said the provision is procedural. The court in this
case says that the context is different: they used the word “procedure”,
but that doesn’t really effect whether or not it is actually procedural for
the purposes of this case.
 The court sees the purpose of the statute as being related to
substantive contractual rights.
 Must consider the purpose of the statute

Alberta Treasury Branches v. Granoff – BC CA (1984) – p. 149

D trying to escape liability by trying to characterize parts of forum law as


procedural – mechanical/formalistic approach

Facts: D executed chattel mortgage in AB, then went to BC. BC statute


limits the rights of mortgagee to exercise contractual rights. Mortgagee
can sue on debt, or seize property and sell it and if there is a deficiency
sue under the debt, but only one or the other. So if they seize it and sell it,
they’ve lost their right to sue for a deficiency. AB doesn’t have that
provision.

Decision:
D says the governing law of the chattel mortgage is the proper law; must
take BC procedure, so can only seize or sue. The BCCA buys this
argument, b/c they say that it speaks to the method of enforcement. If the
statute said D’s rights were only X, it would have been substantive, but
here since it says how they are to be enforced, the provision is procedural.

Black thinks that this is a bad result b/c if you enact consumer protection
legislation as BC did here, lenders will make adjustments for it (e.g. ask
for a greater down payment or charge more interest). The Alberta lenders
aren’t limited by such legislation so they wouldn’t do that. In this case, the
Alberta lenders are suffering b/c they are being treated the same as BC
lenders, and D is gaining through the protections afforded by the BC
statute: really there is no market adjustment occurring so it’s unfair to the
AB lenders. Probably the court wasn’t really thinking about conflict of laws
issues at all.

Block Bros. Realty v. Mollard and Deltra Holdings – BC CA (1981) – p. 152

P is licensed R/E broker in AB claiming commission for land sold in BC.


BC law says a person may not maintain an action in any court unless
licensed in BC. P isn’t licensed in BC, but brings a suit.

Court says law is substantive.

• Consider the consequences of characterization: if prosecution is


prevented by a procedural rule of the law of the forum, then you can’t
even look to the law of another jurisdiction so any c.o.l. rule saying that
the substantive law of another jurisdiction applies will be frustrated.
• A court should not characterize a rule of law of the forum as
procedural just b/c it is characterized as procedural for some
domestic purpose.
• Court also says that legislation should be categorized as
procedural only if the question is beyond any doubt – if it could
go either way, err on the side of substantive.

When thinking about substance/procedure distinction also consider what


the purpose of procedural rules are.

J.D. Almeida Araujo v. Sir Frederic Becker & Co. – Eng. QB (1953) – p. 159

Facts: buyer didn’t pay for goods, seller gets to claim loss on contract, but
b/c buyer didn’t pay, the seller couldn’t buy from his upstream supplier.
Seller’s contract w. upstream supplier had a penalty clause, so now seller
seeks to recover value of contract from the buyer along with the amount of
the penalty clause. The proper law of contract is law of Portugal, but
English court’s rules say that recovery for the penalty clause is too remote.
So D wants English law of remoteness to be procedural so that it will
apply.

H: remoteness is substantive (i.e. governed by proper law of contract)


Court says that some questions of damages might be procedural, but
remoteness isn’t. Dicey says liability for contractual obligations is
determined by proper law, but measure of damages is governed by law of
forum. This makes it so that a party could be deprived of damages he
would have gotten under proper law, just b/c he was lucky enough to get
UK court to take jurisdiction. This harsh effect can me mitigated if the
concept of remoteness is separated from quantification of damages.

- the more rigid the choice of law rule that is used, the greater the need for
escape devices
o for this reason, more applicable in tort cases – rigid rule in
Tolofson

Castillo v. Castillo – SCC (2005) – p. 168

Alberta has legislatively reversed Tolofson:


Limitations Act, R.S.A. 2000, c. L-12, s. 12.

One-car crash in California, w. 2 Albertans in the car. CA has short


limitation period, AB has longer one. Suit is brought in AB, over a year
after the crash. Section 12 of Limitations Act says that AB’s law should
always be applied, but you never get to AB’s longer period b/c Tolofson
says limitation period is substantive and by applying CA’s substantive law,
the shorter limitation period applies. Only AB procedure is used under
Tolofson. This is probably not what the AB legislature intended.

Leave to appeal to SCC has been granted. Black is guessing the SCC will
reverse the interpretations that have been applied up until now, and say
that the statute means not even foreign substantive law will apply.

3. Renvoi
- characterization has occurred, choice of law rule has been applied, governing law
has been selected
- should we ever look at the conflicts law of the foreign law selected?
o Renvoi = making reference to the conflicts rules of foreign law selected
through the choice of law process
o Eventually one court applies a set of contract rules, exclusive of any
contract rules – noone really knows when it stops
 Partial renovoi – over and back once (civil law)
 Total renvoi – three times over (English)
- Renvoi is never used in contracts cases

Re Annesley – Chancery D. (1926) – p. 172

Example of renvoi being applied

Facts: British woman dies in France w. will valid under English law. There is
no limit to testation in English law, but there is in French law: must give a
certain amount of your estate to your children. English law says the law of the
will is governed by the domicile of the testatrix at her death: French law
governs, and is therefore subject to limitation on testation. However, French
choice of law rules say freedom of testation is limited, but only for French
citizens.

Issue: English COL rules point to France, but French COL rules point to
England. Which COL rules should apply?

Decision:
 Court applies renvoi w/o really discussing why it is adopted in succession
even though it doesn’t apply in contract or tort.
 Court applies domestic law + conflicts rules including renvoi rules.
 English court pretending to be the French court looks at English law and
then decides that English law would bounce the question back to French
law, and stops there.
Black thinks this decision is questionable b/c France only wants to apply their
testation law to its citizens; they’d let England apply their domestic law. But
then the English domestic law goes and uses French law that never really
was intended to apply to English citizens.

- Renvoi can be useful in certain situations


- exceptions where renvoi may be applicable
o bequests
o title to foreign land
o some cases involving title to movables
o marriage
- applied to make a bequest/marriage valid not invalid
- general view: renvoi does not apply to tort cases
o Australia has gone the other way

4. Characterization
- bright line distinctions between different areas of law do not exist
- traditional rules assume characterization is possible and consistent
- courts have lots of freedom to decide whether to call something torts of contract
- use characterization to avoid a choice of law rule they don’t want to apply

Haumschild v. Continental Casualty Company – Wisc. SC (1959) – p. 176

Judicial use of characterization to avoid choice of law rules that lead to unwanted
results

WI has spousal immunity rule at this time: spouses can’t sue each other for
torts. Tort law says that l.l.d. applies, and the accident here occurred in CA. D
argues that this isn’t a tort case, it’s a family law case, so WI’s spousal
immunity rule should kick in.

Court applies law of the domicile – characterizes it as an issue of incapacity based


upon family relationship – family law instead of tort

Levy v. Daniels U-Drive Auto Renting – Conn. SC (1928) – p. 178

Connecticut has law where the rental company would be responsible for torts
caused by the operation of its vehicles. Mass. doesn’t have those rules. Is
this a torts case or a contracts case? The court calls this a contracts case.

For these cases Black is saying that we are not really equipped to properly
answer questions of characterization, so it’s pretty much a crap shoot. The
message I’m getting is that if you get this on an exam, you can’t go wrong.
Castel, p. 204 – likes the traditional approach of characterization, and
defends it against the view that we shouldn’t characterize, we should just
look at policy.

5. Public Policy
- if the lex causae – the result selected by the c.o.l. process – is found to be
repugnant, then its application can be declared contrary to public policy
and the court can refuse to apply it
- comes up in the context of enforcement
Part II – Recognition and Enforcement
Chapter 5 – Recognition and Enforcement of Foreign Judgements – pp. 183-308
1. Introduction
- a party who has obtained a judgment in a foreign jurisdiction may be able to have
that judgement given effect in another jurisdiction
- recognition = process by which the foreign judgment is regarded as validly
determining a dispute between the parties
o relevant if the sole issue is the conclusiveness of the foreign judgment
o recognition alone is in issue where foreign judgment is in favour of the
defendant
- enforcement = process by which the foreign judgment is transformed into a
judgment of the forum
- common law - England: foreign judgment creates a debt, you can bring an action
for that debt in the English courts
o criticism of this analysis
o Ont. CA – bring an action on the judgment not the debt; subsequently
rejected in favour of traditional view
o Treated like a contract debt
- Rules re preclusiveness (finality) of foreign decisions are similar to the rules of
res judicata in relation to domestic judgments
- Foreign judgment can be enforced through motion for summary judgment
- Even more efficient mechanisms for enforcement through Reciprocal
Enforcement of Judgments Act
o Common law basis for enforcement remains the same
- Morguard - important case re enforcement of foreign judgments

2. Recognition and Enforcement: Common Law Rules


- common law rules re enforcement of foreign judgments were unchanged from 19th
century until 1990
- 1990: Morguard rewrites the rules
- Not clear if Morguard replaces old rules in every situation
- Old rule
o Rendering court must have had jurisdiction in accordance with the rules of
the enforcing court
 Personal survice in the jurisdiction
 Defendant submitted to the jurisdiction
• Unconditional appearance
• Submission of a defence
• Agreement to submit
• Choice of the forum
o Avoid relitigation, promote fairness & justice – but clearly not every
foreign judgment should be enforced
- Bases for not recognizing/enforcing the foreign judgment
o Asserted jurisdiction in an unfair manner
 Forum non conveniens
 Arbitrary arrest/detention
o Unfair proceedings
 Lack of notice
 No opportunity to present answer to the claim
 Corrupt decision-maker
o Decision was wrong on the merits
 Applied the wrong conflicts law
 Applied the wrong rule of decision
- Canadian approach to recognition/enforcement is narrow
o Did the foreign court have jurisdiction in accordance with our rules?
o The reasoning of the foreign court in taking jurisdiction is not relevant
o Compare the decision reached with our rules – compatible?

Henry v. Geoprosco International – Eng. CA (1976) – p. 192

You have to have submitted on the merits. The question then is how far
you have to have acted in a given jurisdiction before you are deemed to
have addressed the charge on its merits.

F: P says D wrongly dismissed him.


D’s arguments:
- that AB doesn’t have jurisdiction
- that AB has discretion and should decline to take jurisdiction – forum
non conveniens
- this contract has an arbitration clause in it; the whole case should go to
the arbitrator.’
Canadian court rejects these arguments and enters judgment against D.

The English court, when deciding whether or not they will enforce the
judgment, finds that D submitted enough to AB jurisdiction even though he
never argued the case substantively. Even arguing an arbitration clause
can count as fighting the case on the merits.

The standard is not whether or not a party intends to submit, it’s


whether or not the lawyer did enough to count as submitting.

Black-Clawson Int’l v. Papierwerke Waldhof… - House of Lords (1975) – p. 195

Use of a foreign judgment as a defence to an action brought in another


jurisdiction – issue estoppel – recognition only

Facts: Foreign court rules in favour of D. P wants to now sue in NS, and D
wants NS courts to recognize foreign ruling and not allow P to sue again in
NS.
Issue: When using a foreign judgment as a defence you’re talking issue
estoppel, b/c the foreign judgment can’t be regarded as res judicata.
There is no certainty that when a court finds in favour of a defendant that
the case has been decided on its merits. On the one hand, a court could
fully adjudicate a case on its merits and then dismiss the action, but on the
other hand a court could dismiss an action based on a technical issue,
such as deciding it is not the proper forum to deal with the cause of action.

Discussion:
In this case the German court dismissed the cause of action b/c it was
barred by limitation period. Is this a sufficient reason to not allow P to sue
again? Ger. has short limitation period, but views limitation periods as
being substantive. German law does the renvoi and says that English
limitation period should apply; English law says limitation period is
procedural and bounces back to Germany (I don’t even know if I put this
down right).

H: P gets to pursue the claim b/c it wasn’t decided on the merits before

De Savoye v. Morguard Investments Ltd. – SCC (1990) – p. 198

The single most important case of this course – rewrites the rules of
enforcement in Canada

Facts: D gets mortgage on house, Calgary housing market takes a dive,


and D walks away from the house. P sells it, but there is a deficiency, so
bank sues on deficiency judgment. Bank serves D in BC, where he
moved. D ignores service: hadn’t submitted in any way, and he wasn’t
present in AB when served. [the judgment would always be enforceable in
AB, so D can never really go back there, but D is OK in BC]. The SCC
says this is BS.

This is the single most important case we’ll look at; has consequences for
a lot of other stuff.

Decision:
 Under the old rules (presence, submission) this would not have been
enforceable
 Court broadens the old rules using the concept of reciprocity.
This can mean two things: (1) we’ll enforce your judgments if you
enforce ours; or (2) how domestic jurisprudential practice works: would
we take jurisdiction in like circumstances: e.g. if the same thing
happened in BC, would they do ex juris service and hold D
responsible. Another possible basis for jurisdiction is nationality, but
that’s not good law and it doesn’t really apply any more (bottom of p.
224).
 SCC thinks the conception of comity that underlies the traditional
enforcement of foreign judgments needs to be adjusted in light of a
changing world order: we should have ready enforcement of
judgments across borders; this is necessary to facilitate trade:
“accommodating the flow of wealth, skills and people across state lines
has now become imperative.”
 More importantly though, it was a mistake right from the beginning to
transpose 19th century rules applying to judgments from foreign states
into rules enforcing judgments from sister-provinces. The
considerations underlying comity apply w. more force btw. the
units of a federal state.
 There are a number of things that make the federal interprovincial
scene different from the international scene:
o Charter, s. 6: mobility rights – people can move, so should
judgments
o Constitution Act, 1867, s. 121 - Common Market
o Constitution Act, 1867, s. 91(2) - federal Trade & Commerce
Power
o Federal POGG power
o Superior court judges are appointed by federal authorities
 For these reasons Canada doesn’t need a “full faith and credit” clause
in the Constitution like the US does, which requires that all states
enforce each other’s judgments. The Australian Constitution gives the
federal government power to address the issue of inter-state judgment
enforcement, which it has done. The EC has also moved towards
automatic judgment enforcement: because they have a common
market, they have also decided that there is no defence to a judgment
of an Italian court sought to be enforced in England.
 There should still be a jurisdictional test though:
o Order and fairness - requires that a person not be able to avoid
obligations just by moving to another province (fairness to Plaintiff)
o Real and substantial connection – the court giving judgment
must have properly exercised jurisdiction (fairness to Defendant: P
can’t go to any court to sue).
 So you can no longer do the flip that the defendant attempted in this
case: if AB has a real and substantial connection with the cause of
action the judgment can be enforced. This adds a huge option:
presence and submission are still options, but now also a real
and substantial connection is as well – assumed jurisdiction

Important Obiter: Service Ex Juris


La Forest also says we should consider using the Constitution to limit
provincial service ex juris rules. If the courts of one province are expected
to give effect to judgments given in another province, there must be some
limits to the exercise of jurisdiction against persons outside the province.

The US has its full faith and credit clause, but it doesn’t have the authority
to enforce broad service ex juris: there are limits on long-arm jurisdiction.
Similarly in Australia, there are no bars to judgment enforcement, but
there are limits to service ex juris so that provinces can’t just bring
anybody before their courts. The Brussels Convention (EC) also sets out
the maximum service ex juris rules.

If you have enforcement, provinces can’t take jurisdiction over


foreign defendants in any instance they want: there’s a relationship
between the original assertion of jurisdiction and then enforcement of
foreign judgments.

So La Forest in Morguard makes the very important argument that not


only should we allow more enforcement of judgments,
constitutionally we should limit the legislation that purports to give
original jurisdiction to hear cases in the first place.

Note:
Morguard left it open as to whether the real and substantial connection
rule also applies to extra-national judgments. La Forest pretty much builds
his arguments based on interprovincial reasons. But certain cases have
held that Morguard does apply to foreign judgments. (see note for cases)

Minkler and Kirschbaum v. Sheppard – BCSC (1991) – p. 213

Applying Morguard to int’l judgment; judgment could have been a lot


different if rendered in Canada.

Note:
The substantial connection test is unclear as to what there must be a
connection between:
o subject matter of the action and the territory in which it’s
brought
o damages suffered and the jurisdiction which rendered the
original judgment
o the rendering jurisdiction and the defendant
o sufficient “contacts the rendering jurisdiction may have to the
defendant or the subject matter of the suit”

Note:
After Morguard most defendants raise a defence at the jurisdiction stage.
There are two options for battling a suit commenced in another
jurisdiction:
i) Go to the other jurisdiction and say the court has no jurisdiction
to hear it
ii) Wait until judgment is issued against D, and then when P comes
to D’s province to enforce it, say there was no real and
substantial connection

Braintech v. Kostiuk – BCCA (1999)

R&S connection test not satisfied in online defamat’n

Facts: One of the first internet cases in Canada. Braintech is in Texas or


something, Kostiuk is in BC, and he posts something online that B finds
defamatory. B sues K in Texas, and then tries to get the Texas judgment
enforced in BC.

Decision:
 BCCA finds that there was no real and substantial connection between
the cause of action and Texas, so they won’t enforce the judgment.
 D hadn’t even shown that anyone in Texas had even read the
defamation.
 An argument for thinking this holding is correct is that if you allow a
court to take jurisdiction just b/c the web site w. the defamation is
available online in Texas, then any court worldwide could take
jurisdiction. There should be a more substantial connection than that.
 Dubious argument: mode of service.

Provinces can legislate away from Morguard re: international


judgments.
Sask. and NB have statutes that require pre-Morguard rules to
enforcement of international judgments. These don’t apply w/in Canada
b/c they wouldn’t be constitutional. Quebec also has tougher standards.

- Morguard – recognition that conflicts cases raise constitutional concerns


o Increased ability to bring in non-resident defendants
- If the assertion of jurisdiction was approved by the Supreme Court, then a
province would have a hard time arguing that it shouldn’t be enforceable

3. Judgments that will be Refused Enforcement


- old rules: additional bars to enforcement besides jurisdictional issue
- uncertain to what extent these are applicable post-Morguard

(a) Public Policy


- court has the power to exclude offensive claims in order to prevent an abuse
of its process
o refuse to listen to argument or give a remedy on the basis of public
policy
o ‘impeachment’ defence

Boardwalk Regency Corp. v. Maalouf – Ont. CA (1992) – p. 227

At c/l a foreign judgment won’t be recognized or enforce in Canada


if it would be contrary to public policy to do so.

Not every legal prohibition raises a public policy defence. Issue in


this case is whether the language of the Gaming Act, apart from its
impact on domestic contracts, is an expression of public policy such
that it would be offensive for a court to participate in enforcement of
foreign judgment.

Ontario’s law against enforcing gambling debts is not based on


public standards of morality and therefore a gambling debt arising
out of a contract in NJ is not unenforceable in Ontario on the
grounds of Public Policy.

Beals v. Saldanha – SCC (2003) – p. 230

substantial connection test applies to foreign country judgments;


status of public policy defence

Possible arguments about impeachment defences:


i) Morguard doesn’t effect impeachment defences:
jurisdictional requirement and impeachment concerns are
unrelated separate matters in a case.
ii) We should trim back impeachments defences even more,
b/c Morguard emphasizes the importance of allowing foreign
judgments to be enforced in the name of comity and
reciprocity
iii) We should broaden impeachment defences: we should
rethink the whole package; we’ve never really defined the
impeachment defences, and should do so in light of the fact
that since before Morguard the defences were construed
narrowly, now that enforcement has been broadened, we
should broaden defences as well.

Facts: Ds defend one action and are successful, they defend a


second action, but don’t defend subsequent amendments, which
are required in order to maintain defence. Eventually judgment is
issued in Fla. to the tune of $270,000, at which point they consult a
lawyer, who tells them to ignore it b/c they haven’t attorned to the
jurisdiction (even though this is one year after Morguard so the real
and substantial connection applies). Sideline here is that b/c of the
lawyer’s negligence it is his insurance company that is on
Saldanha’s side (?) taking this all the way to the SCC.

Decision:
 After Morguard we shouldn’t change our concept of
impeachment defences

Fraud: Narrow
 If you allow a broad defence of fraud it will be troublesome
 Fraud going to jurisdiction can always be raised as a defence
 The merits of a foreign judgment can only be challenged for
fraud only where the allegations are new and not the subject of
prior adjudication
 Evidence of fraud has to be new and material, undetectable by
foreign court.

Natural Justice: Unfair Procedure


 Enforcing court must ensure D was granted a fair process: Fla.
judgment fell below Ont. standards, but how different does it
have to be?
 Fla. claim didn’t say the amount, only “in excess of $5000” – no
indication that it could involve treble damages, punitives, etc. to
amount to over $270,000 to account for P’s claim that the model
home they were going to build on the lot was going to make
them a lot of money
 Ds didn’t realize that they had to put in a new defence for each
amended statement of claim
 Majority holds that you can’t put on Ps the obligation to educate
Ds about differences in procedure

Public Policy: Unfair Law


 This defence should be applied narrowly: only injustices
that offend our sense of morality
 The high damages awarded by a Fla. jury, while high, don’t
offend our sense of morality.

Dissent:
 There is particular concern w/r/t default judgments…
Natural Justice:
 If foreign procedure is different enough to result in an excessive
judgment against D, there should be an obligation on P to
educate D on differences that may give rise to the harsh result.
Public Policy
 LeBel suggests we shouldn’t enforce US punitive damages
awards in cases where we wouldn’t enforce them. We don’t
know on the facts whether the Fla court bought the argument
that the Beals were malicious and high handed etc.
 Maybe Cdn. courts should enforce the judgment but trim down
the amount of punitive damages.
 The majority didn’t really address public policy on the punitive
damages.

(b) Foreign Tax Claims


- foreign tax claims won’t be enforced and courts will refuse to take jurisdiction
– ‘revenue rule’
- many countries have bilateral tax treaties that provide for reciprocal cross-
border enforcement though

(c) Foreign Penal Claims


- foreign judgment for a fine will not be enforced
- area of corporate legislation brings borderline issues – civil or penal?

Huntington v. Attrill – Privy Council (1893) – p. 264

Facts: H lent money to NY company of which Atrill was director. Under NY


law, any misrepresentation signed by officers of a company made the
officers jointly and severally liable for any damage coming from the
misrepresentation.

Held:
The standard of recovery wasn’t a fine. The measure was the harm to
creditors (i.e. plaintiff’s loss), not the government, so this isn’t a claim “in
the nature of a suit in favour of the state whose law has been infringed.”

(d) Foreign Judgments Concerning Land


- Canadian courts may not enforce a foreign judgment dealing with Canadian
land

Duke v. Andler – SCC (1932) – p. 267

Facts: Ps and Ds both lived in CA. Ps sold BC land to Ds, who breached
agreement of purchase and sale. P sued to get land back and CA court
ordered a reconveyance back to P. Ds refused to convey, so P brought
suit in BC.

Held:
- Can’t enforce foreign judgments dealing w. title to land in
province.
- Parties in CA would have to come to BC and relitigate.
- SCC says even if they knew CA had applied BC law they still wouldn’t
enforce the judgment
- The rule here is also that money judgments based on title to land
shouldn’t be enforced either. It’s not just in rem rulings aren’t
enforced, it’s that anything involving a foreign finding of title isn’t
enforced.

This doesn’t arise in Canada b/c, e.g. NS won’t take jurisdiction to give
title to land in NB.

4. Enforcement of Non-Monetary Awards


- how should we deal with foreign non-money judgments?
o Declarations
o Injunctions
o Specific performance of a contract

Pro Swing Inc. v. Elta Golf Inc. – SCC (2006) – p. 270

- sets out rules re foreign non-money judgments


- balance national identity and jurisdiction vs. globalization & increased
mobility of people/assets
- court was asked to change the common law
o common law: prevents enforcement of non-money judgments
o changes must be made cautiously
o we need a new rule, but this is not the case to implement it
- the court recognizes the need to change the rule, but because of problems with
the particular orders in this case they don’t do it here
o order is quasi-criminal, etc.
- Dissent
o Similar principle to majority, but different result
o They would enforce the judgment

5. Statutory Provisions and International Agreements


(a) Provincial Statutes
- provincial jurisdiction to legislate re enforcement of foreign judgments:
‘Property and Civil Rights in the Province’
(i) Reciprocal Enforcement Legislation

 In force in all the provinces


 Product of Uniform Law Conference
 The statutes provide a cheaper, quicker way of enforcing judgments
that could already be enforced at c/l but not allow for judgments that
wouldn’t be enforced at c/l:
2(1) the judgment creditor can apply to a court for an order for the
judgment to be registered. So the Act just helps save the time and
expense of having to bring an action

The Act contains all the pre-Morguard defences though:


3 The judgment won’t be enforced if:
(b) The judgment debtor wasn’t carrying on business or residing
w/in jurisdiction of original court and didn’t voluntarily appear or
otherwise submit to proceedings
(c) The judgment debtor wasn’t duly served and didn’t appear or
otherwise submit to jurisdiction of the court
(d) Obtained by fraud
(e) An appeal is pending or judgment debtor intends to appeal (at
c/l the judgment has to be final)
(f) Public policy says it shouldn’t be enforced
(g) The judgment debtor would have had a good defence if an
action were brought on the original judgment

This is out of synch w. Morguard. The next statute accounts for that:

The drafters intentionally stayed on the conservative side.

Uniform Enforcement of Canadian Judgments Act

 Doesn’t depend on reciprocity – one province that enacts it will have to


enforce judgments of another province even if that other province
hasn’t enacted it.
 “judgment” is a final judgment of a Canadian court
 Morguard drew lines between Canadian judgments and other
judgments; and this Act follows that decision

Only defences are in 6(1):


Court can stay or limit enforcement of a registered Canadian judgment if:
a) An order could be made by superior court under its rules or
enactment of enacting province relating to creditors’ remedies and
enforcement of judgments
b) Appeal: the judgment debtor brings or intends to bring a
proceeding to set aside or vary the original judgment
c) An order staying or limiting judgment is in effect where
judgment was made
d) Judgment is contrary to public policy

This is fishy b/c it lacks the “real and substantial” connection from
Morguard.
It’s not a reason for an NS court that passes the statute to make an order
limiting another province’s judgment on the grounds that the other
province doesn’t have jurisdiction…

Drafters of the statute thought that it’s not a good policy to allow
defendants to lay low when the case takes place elsewhere and then
fight it when judgment creditor comes to defendant’s jurisdiction to
enforce the judgment on the grounds that there’s no substantial
connection. Allowing people to do that is bad policy b/c it could cause a lot
of wasted time and money if P is successful in foreign jurisdiction but then
cannot get another jurisdiction to enforce it. 6(1) essentially requires
someone to show up in the foreign jurisdiction at the jurisdiction
stage rather than waiting for the enforcement stage.

There might be an argument that, b/c Morguard brought the constitution


into play, this law if enacted would be unconstitutional b/c it takes away a
defendant’s right to stay at home. If Morguard says that a BC court can’t
hear a case if there is no real and substantial connection, it would be
unconstitutional to require a Nova Scotian to go to BC to fight a suit there.
The argument is that 6(1) should make reference to “real and
substantial connection”

(ii) Arbitration Awards


- arbitration awards are enforceable under Reciprocal Enforcement Act in the
same manner as a court judgment
- international arbitration requires minimal state interference
- United Nations Convention
o Attempt to balance state’s domestic interests with the need for neutral
enforcement of arbitration awards
o Implemented in Canada by federal/provincial governments

Schreter v. Gasmac Inc. – Ont. GD (1992) – p. 296

Application to enforce a foreign arbitration award in Ont. under the


International Commercial Arbitration Act

H: Ont. court recognized it.

(iii) Other Provincial Legislation


- Motor Vehicle Acts provide for extraterritorial application of a license
suspension
- Insurance Acts: attempts to alleviate the strictness of pre-Morguard
enforcement

(b) Canadian Statutes and International Agreements


- Federal Court established by Federal legislation – enforceable across Canada
o Court has very limited jurisdiction
- Discussion of Foreign Extraterritorial Measures Act
o in response to broad American assertions of jurisdiction. Gives
capacity to AG to make an order that a foreign judgment not be
enforced where the recognition or enforcement of it would
significantly interfere w. significant trade and commerce or
Canadian sovereignty (s. 8). Also s. 5 allows AG to prohibit a
Canadian from complying w. a foreign order; e.g. US legislation
tries to tell US companies to get their subsidiaries to not do
business w. Cuba, so AG says ignore it.
Part III – Jurisdiction
Chapter 7 – Jurisdiction of the Provincial Courts – pp. 355-523
1. Introduction
- in any proceeding, the defendant can argue that the court lack jurisdiction to hear
the case
- rules re jurisdiction
o statutes
 i.e. Divorce Act
o common law
 rules re contract, tort and property
 more complex
 deal mostly with judgments in personam, and not in rem
- courts of general jurisdiction – original common law courts in each province
o defendant has to show that the court should not hear the case
- other courts have limited jurisdiction
o plaintiff has to convince the court to hear the case (i.e. leave to appeal to
SCC)
- close relationship between jurisdiction and choice of law
o the fact that courts take jurisdiction in geographically complex cases is
what necessitates choice of law rules
o underlying factors should be essentially the same
- all jurisdictions are limited geographically; laws were designed for application
inside the jurisdiction – no geographically complex facts
- focus on the criteria considered by the court
- basic position: jurisdiction of any court is territorial
o idea of sovereign ordering someone to appear, have to be subject to the
sovereign – sovereignty traditionally seen as territorial
o citizenship was developed later
- anyone who is served in accordance with the Rules of Civil Procedure is subject
to jurisdiction
o generally requires personal service
o corporations present unique problems
- four general problems created by geographically complex facts
o are the rules that disable from hearing the case?
o Should the court refuse to hear a case where the defendant is properly
served as a matter of justice/fairness?
o When is a court justified in hearing a case with a defendant outside its
borders?
o When will a court enjoing the parties from continuing proceedings in a
foreign court?

2. Rules Excluding the Jurisdiction of the Courts


(a) Personal Exclusions
- principal exclusion: sovereign immunity
o cannot assert jurisdiction over foreign sovereigns
o applies to jurisdiction & enforcement
- legislation: State Ammunity Act(federal), also provincial
- leading case on sovereign immunity: Re Canada Labour Code

Bouzari v. Iran – Ont. Sup. Ct. Jus. (2002) – p. 359

Facts: Bouzari used to be a consultant to oil and gas companies, helping


them get lucrative oil deals w. the Iranian government. He refused to give a
bribe to an Iranian official, so he was imprisoned and tortured. After paying
about $3 million in fines he was dumped, blindfolded, in the middle of a traffic
circle in Iran. Now he’s in Canada

Discussion:
- Bouzari has no real and substantial connection to Canada (Tolofson rule),
but court doesn’t rely on that fact in dismissing his case.
- State Immunity Act
o Doesn’t apply to crimes; P argued that his case was penal in
nature, but court disagreed: still a civil suit therefore not criminal
proceeding.
o S. 6 exception to commercial activity: P argues that b/c he had to
pay bribes in relation to an oil deal, Iran’s actions were commercial
 Nature: police and prison activity not commercial in nature at
all
 Purpose: sort of commercial
Although the SCC, has previously held that both nature and
purpose can be considered, here the acts of Iran weren’t even
close to being commercial when all things are considered.
o S. 5 exception to torts: this applies only to torts committed in
Canada
- w/r/t/ P’s argument that exception should be read into State Immunity Act,
SCC has held that you can look to int’l norms to resolve ambiguities in a
statute: court looks to state practice, and finds that no other courts in the
world had ignored state immunity in order to allow civil suits in torture
cases.
- Charter argument doesn’t fly either…
- A decision on excluding immunity for torture was left to the legislature

(b) Subject Matter Exclusions


- power of provincial courts to hear cases is limited by a number of factors
o historically, some provinces couldn’t issue divorce
 solved by federal Divorce Act
o federal legislation may give another court jurisdiction
 i.e. Federal Court re admiralty, federal taxation, etc.
o can’t hear a claim regarding foreign land
 may not apply inter-provincially
Hesperides Hotels Ltd. v. Muftizade – House of Lords (1979) – p. 378

Arguments by P:
o This isn’t trespass, it’s conspiracy to trespass
o There should be an exception where there is no dispute as to claim
of title. H.L. finds that there is no such exception: the original rule
generalized and didn’t allow room for distinguishing cases where
there is no dispute so that can’t be an exception to the rule
o There are exceptions to this subject matter exclusion: you can sue
a ship that runs into land and damages it.

In Godley v. Coles (1989) Ont. Dist. Ct. the judge decided not to apply the
Mocambique Rule. This is pretty exceptional, and it was done because to
apply Mocambique just wouldn’t make sense.

After Morguard there is an argument that Mocambique shouldn’t apply


interprovincially, but this argument hasn’t won any cases yet.

- Mocambique rule
o No jurisdiction re:
 Determination of title to foreign land
 Trespass on foreign land
o Exceptions
 Court has jurisdiction in personam due to contract, trust or
partnership
• Case where action for unjust enrichment met the in
personam exception – Minera Aquiline Argentina v. IMA
Exploration – BC SC (2006)
 Administering an estate or trust that includes property in Canada,
as well as foreign land
 Jurisdiction in rem against a ship
o Lower courts are sometimes reluctant to apply Mocambique strictly
o Potential for analogy between case dealing with land and copyright cases

3. Other Restrictions on Taking Jurisdiction


(a) Forum Non Conveniens
- serious concerns about forum shopping
- this a rule that limits the plaintiff’s freedom to choose the forum
o courts will control an unfair exercise of this freedom – abuse of process
o basis for interference has been expanded in recent years
- principal issue: should the court be plaintiff or defendant-oriented
o initial position: defendant has to show that the choice of forum was
vexations and an abuse of the court’s process
Moreno v. Norwich Union Fire Insurance Society – Ont. HCJ (1971) – p. 390

Example of Traditional Forum Non-Conveniens

F: Norwich insured Collins, a US air force pilot serving in UK. Collins killed
Moreno, so Mrs. Moreno sues Collins, who is found liable in Illinois court
and so assigns his interest in insurance policy to her. Norwich won’t pay
under policy (says Collins drunk), so M sues Norwich in Ont., who claims
the Ont. court is Forum Non-Conveniens (FNC) b/c action brought by
parties who live in US arising out of MVA that happened in UK involving
an insurance policy to be interpreted under law of UK would be unjust and
an abuse of court’s process.

H: Ont. Court is FNC

R: Although a court will not lightly interfere w. a plaintiff’s chosen


jurisdiction, if the inconvenience of trying a case in a certain jurisdiction
would be a serious injustice to defendant and be vexatious then the
court can use its discretion to decline to hear the case.

o this restritive test has been relaxed in recent English cases


 balance of convenience test

Spiliada Maritime Corp. v. Cansulex Ltd. – House of Lords (1986) – p. 394

FNC in Canada

F: P’s ships were damaged in BC, and owners of the Spiladia were
Liberian, managed partly in Greece and partly in England. P obtained
leave for service ex juris; suit brought in UK. D argues FNC, but P says
they’d lose a big juridical advantage by not being able to sue in UK b/c suit
in BC would be statute-barred.

H:
• P must first have founded jurisdiction as of right in the country where
the action is brought.

• In order to justify a stay two conditions must be satisfied (one positive,


one negative)
i) The defendant must satisfy the court that there is another
forum to whose jurisdiction he is amendable in which justice
can be done at substantially less inconvenience or
expense.
ii) The stay must not deprive the P of a legitimate personal or
juridical advantage which would be available to him if he
invoked the jurisdiction of the forum in which he brought the
claim.

• Court distinguishes between jurisdiction as of right and service


ex juris – the standard for finding FNC is lower for service ex
juris. For (i) above, effect is that in service ex juris cases, despite the
BOP (below) the P will have to show that it’s not an inconvenient forum
b/c there will be so much expense and inconvenience to D.

o Burden of Proof:
 On D to persuade the court to exercise its discretion to
grant a stay
 If D establishes prima facie that another forum is more
appropriate, BOP is on P to show special circumstances
that justice requires trial should still take place in the forum in
which he brought the claim.

o Losing Juridical Advantage: the fact that lower damages would


be awarded in the more convenient jurisdiction doesn’t count – a
court should not be deterred from granting FNC as long as
substantial justice can be done tin the more appropriate forum.
o Note: this case closely connected to Amchem
o This case has been approved by SCC – may be different
interprovincially

(b) Contractual Limits on Forum Selection


- international contracts will usually specify a governing law and choose a
forum for litigation/arbitration
- two kinds of forum clauses
o agree to submit to a particular jurisdiction but don’t specify that this
is an exclusive jurisdiction
o confer exclusive jurisdiction on a specified court
- non-exclusive clause
o makes the court competent to hear
o makes judgment enforceable pretty much anywhere
- exclusive clause
o courts don’t like it
o won’t enforce an exclusive clause in an unfair contract
o otherwise, they’re stuck with it
- court’s discretion in controlling forum selection clauses – common law -
Eftheria
o court is not bound to grant a stay, discretionary power to do so
o stay should be granted unless there is strong reason not to
o take into account all the circumstances in the case
 location of evidence
 applicable law, connections between the parties and the
jurisdictions
 are the defendants just seeking procedural advantages?
 Would the plaintiffs be prejudiced by having to sue in the
foreign court?
o Common law rules were unpredictable and unsatisfactory – litigants
need to know the relevant criteria a court will apply
- Canadian courts usually give effect to choice of jurisdiction clauses

Z.I. Pompey Industrie v. ECU Line N.V. – SCC (2003)

Appeal from Federal Court judgment refusing to give effect to choice of


jurisdiction clause

Facts: contract to ship goods from Europe to Seattle said not to ship by rail
and included exclusive jurisdiction clause in favour of Antwerp. Shipper
shipped by rail, breached contract. Action was started in Cdn. fed. court,
which decided to exercise discretion and not grant a stay based on the
exclusive jurisdiction clause, using the same tripartite test that is used when a
court is asked to issue an interlocutory injunction.

Decision:
 SCC overturned fed. court, confirming the “strong cause” test in
Eleftheria. Discretion still exists, but should only be used in extreme
circumstances.
 Parties need contractual certainty
 Lower court used fundamental breach as an excuse for ignoring the
exclusive jurisdiction clause

Courts have discretion to ignore exclusive jurisdiction clauses in


contracts, but should use it only in extreme circumstances. The “Strong
Cause” test from Eleftheria applies.

Black:
Why not make the rule not discretionary, but rather have a rule that an
exclusive jurisdiction clause can only be ignored for a reason already
recognized in contract law that allows courts to ignore a clause in a contract
(e.g. unconscionability, duress, fraud).

Even the things taken into account in the test don’t necessarily relate to
freedom of contract, e.g. why would the court take into account where the
witnesses are? The parties may well have known or taken into account the
place of the witnesses when they made the contract.

4. Service Ex Juris
- aka ‘long-arm’ jurisdiction
- significant change to common law jurisdiction
- first introduced so that leave had to be obtained for any service ex juris
o convince the court this is a proper case to authorize this exceptional power
o courts will be exceedingly careful
- recent expansion in the court’s powers: three types of rules
o narrowest: service ex juris only if it fits into a specified category (Alb. &
NL)
o similar to the first, but with a catch-all provision giving discretionary
power in instances where specific categories aren’t met
o Nova Scotia – service without leave anywhere in Canada or US; service
outside Canada or US only with leave

Moran v. Pyle National (Canada) Ltd. – SCC (1975) – p. 422

Deciding where a tort occurs for purposes of service ex juris rules.

Facts: D was Ont. company that made light bulbs. P lives in Sask., where
D’s negligently manufactured bulb killed P’s husband.

Issue: is service actually authorized by the rules?

Decision:
o All constituent elements of the tort must occur in the forum in order
for an action to lie. The court considers whether the tort occurred in
Ont. or Sask. Approaches:
 Place of acting
 Place of injury
 SCC likes Distillers Co. v. Thompson where the P.C. hints
at a real and substantial connection test. It rejected a
mechanical application of “last event” theory in favour of a
more flexible qualitative and quantitative test. It held the tort
had occurred in Australia b/c there had been a failure to
warn of possible damage. This failure was a continuing act
that had been made in Australia, and that combined w. the
damage in Australia ended up w. finding for P.
 The defendant’s act which gives the P his cause of
complaint must have occurred w/in the jurisdiction.
There can be no liability until the damage has been
done.
o SCC concludes: for the purposes of servis ex juris rules, a tort
occurring anywhere that D could reasonably foresee it is fair
game. If you can anticipate harm occurring there, you can
have committed the tort there.
o Although this test is pretty broad, it doesn’t mean that, e.g. every
damage that occurs in Sask. will allow for an action.
o You can see Morguard as a constitutionalization of Moran v. Pyle:
it’s reasonable for Sask. to take jurisdiction here; how could you
then say the judgment isn’t enforceable? Of course, Morguard took
place 15 years after Moran.

Where a foreign defendant carelessly manufactures a product in a


foreign jurisdiction which enters into the normal channels of trade and
he knows or ought to know both that as a result of his
carelessness a consumer may well be injured and it is reasonably
foreseeable that the product would be used or consumed where
the P used or consumed it, then the forum in which the P suffered
damage is entitled to exercise judicial jurisdiction over that foreign
Defendant.

Robinson v. Warren – NS SC Appeal (1982) – p. 429

NS service ex juris rules allow service anywhere in North America so


issues like the ones in Moran v. Pyle don’t arise. Forum Non-Conveniens
becomes much more important though:

P was passenger in D’s car, injured in accident in AB. P returned to NS


and served D in AB. D said P’s claim should be struck out on FNC
grounds.

H: For P: P’s action in NS not frivolous or vexatious to D. P lives in NS and


that’s where medical experts are. Appeal dismissed. Long quote from
Catel’s Conflict of Laws is pretty good. Things might be different now that
constitutional considerations are more prevalent (Morguard – R&S
connection)

Note on Actions and Judgements In Rem


- in rem: binds the parties to the legal relation established in the judgment and also
binds third parties
o admiralty
o divorce
o any proceeding that decides title

5. Anti-Suit Injunctions
- a form of ‘offensive’ forum non conveniens
o enjoins someone from starting or continuing an action in another
jurisdiction
- became common in the last few years

Amchem Products v. British Columbia Workers Comp – SCC (1993) – p. 433

3rd out of 4 big cases in 1990s (Tolofson and Morguard are first two we
covered)

Facts: A bunch of plaintiffs, mainly in BC, were injured by working w.


asbestos and sue about 30 asbestos cos. in Texas. Ds get anti-suit injunction
from BCSC; Ps get anti-anti-suit injunction from Texas court. In order to get
anti-suit injunction you have to have an action going in BC so remedy Ds
seek as Plaintiffs in BC action are damages for abuse of process (w. trial
judge dismissed) and a declaration that they are not liable to claimants w/r/t
any of the matters raised by Ps in Texas action. Negative declarations such
as this are not usually issued, but this is a case where one could be.

Issues:
Is Texas an appropriate place to sue, since there isn’t an obvious home state
for the companies? Is it appropriate for BC to issue an anti-suit injunction
against BC Plaintiffs?

Decision:
 Texas took jurisdiction b/c it was as good as any forum to hear the case.
 Texas doesn’t have forum non conveniens (that changed after this case –
FNC is allowed); can take jurisdiction over any defendant who does
business in Texas. It is limited only by the due process clause of the 14th
Amendment of US Constitution.
 Lower Court: found that b/c Texas doesn’t have forum non conveniens
they were an inappropriate forum: P’s action there is oppressive or
vexatious. Issued anti-suit injunction
 Spiliada forum non-conveniens test:
o Identify more appropriate forum. If it is elsewhere,
o Stay proceedings unless it deprives X of significant juridical
advantage
 Sopinka modifies Spiladia for ASI purpose:
o Consider Both Steps Together: juridical advantage should be
weighed with other factors in determining which forum is more
appropriate.
o Burden of Proof from Spiladia:
 When service is as of right (i.e. service w/in jurisdiction)
onus is on defendant to prove forum non-conveniens;
 When service is ex juris, onus is on plaintiff to prove no
FNC (in UK P has to bring motion to get court’s permission
to do service ex juris). Canada has service ex-juris as of
right;
 Sopinka decides that the rules of court dictate who has
onus: if rules of court of a province allow service ex
juris then onus is on D to prove FNC.
 Anti-Suit Injunctions:
o Initially fairly lax: Bushby v. Munday (1821)
o Later it was determined to be the flip side of forum non-conveniens
test: Castanho v. Brown and Root
o SNI realized that ASI was not the flip side of FNC, because it
effectively interferes w. foreign jurisdiction. Held that anti-suit
injunction should only be issued when foreign proceedings were
vexatious or oppressive.

Procedural aspects of ASI in Canada:


- Never get an anti-suit injunction until action is started in foreign
court
- Must give foreign court a chance to decline jurisdiction
- Would it make sense for the forum jurisdiction where the ASI is
sought to hear the application?

Anti Suit Injunction Test:


Is there another forum more clearly appropriate? E.g. Is BC the natural
forum?
If there is no other forum more appropriate, would it be an injustice
for case to be heard in foreign jurisdiction? E.g. If BC is the natural
forum, would it also be an injustice to allow Texas to hear the case?

 In this case asbestos cos. don’t pass first step of the test: just b/c Texas
didn’t use FNC doesn’t mean they wrongly took jurisdiction. Many
jurisdictions don’t use FNC, and Texas is limited by provision in 14th
Amendment in US Constitution – look at the decision, not the process
used to reach the decision

Recent HL case held that ASI should only be issued to protect domestic
jurisdiction. They declined to issue one when India was argued to be natural
forum and Ds wanted vexatious proceedings to be stopped in Texas. UK
didn’t want to get involved.

6. Mareva Injunctions
- Lister v. Stubbs – plaintiff may not get an interlocutory injunctions before
trial that would prevent defendant from dealing with its property in a way
that might deprive plaintiff of an effective remedy
- Mareva is an exception to this general rule
Mareva Compania Naviera v. International Bulkcarriers – Eng. CA (1975) – p.
450

In appropriate cases court can issue injunction to D to freeze assets.

P must show:
1. Reasonable cause of action (don’t have to show an indisputable claim)
2. Real likelihood that D will take money out of the jurisdiction (don’t
necessarily have to show that D has done so in the past)

Considerations:
 D does not have to be outside of the jurisdiction for a Mareva injunction to
be issued.
 Injunctions are effective against 3rd parties with notice: e.g. if D disobeys
Mareva injunction and then skips town so that he is out of reach of court,
the injunction binds not only D but also other parties that have notice (e.g.
banks).
 Injunction can also apply when there is fear of dissipating assets rather
than moving assets out of jurisdiction
 Mareva is an exceptional remedy b/c of danger of its abuse (tying up
assets of D to pressure to settle etc.)

Aetna Financial Services v. Feigelman – SCC (1985) – p. 462

F: Aetna had office in Toronto and head office in Montreal; had an office in
MB. Feigelman went into default under some debentures held by Aetna so
they appointed a receiver for them. F now wants a Mareva injunction to keep
Aetna from transferring assets to its offices outside MB.

I: Can Mareva injunctions be issued inter-provincially?

H: Provincial courts shouldn’t be as concerned about effects of Mareva


injunctions on bank managers in other provinces. Taking into account
constitutional considerations a la Morguard.

• In the federal system, out of province D is not a foreigner; isn’t outside of


jurisdiction for the purposes of a mareva injunction: any judgment obtained
in a province can be executed in another
• Here no clandestine transfer of assets intended to defraud P or make D
judgment proof – Aetna publicly announced its intention to transfer the
assets.
• Therefore UK Mareva rules don’t transfer intact to Canada

United States of America v. Friedland – BCSC (1996) – p. 466

- issue of stand-alone Mareva injunctions in aid of proceedings brought elsewhere


- interlocutory assistance can be granted to proceedings in a foreign court without
an underlying cause of action

7. The Constitutional Component


- Canada is a federal state, different that England (unitary state) where most of our
rules have come from
- National perspective on service ex juris is very important
- Need a vision of jurisdiction that accords with constitutional imperatives

Hunt v. T&N – SCC (1994)

Fourth big SCC case, confirmed the constitutional aspects of jurisdiction


proposed in Morguard.

Facts: Hunt worked w. asbestos in BC and got cancer. Ds relied on Quebec’s


Business Concerns Records Act, which was originally devised to protect
Canadian companies from US anti-trust suits and says that it’s against the
law for businesses to send documents out of the province. Ds therefore
refused to comply w. discovery in BC suit. At trial BC courts deferred to
Quebec Act. Now P raises constitutional challenge: BC should ignore Quebec
legislation b/c it’s unconstitutional.

Issues: (i) Is Quebec’s statute ultra vires its powers in ss. 92(13), (14) & (16)
of the Constitution Act? (ii) Does Morguard make the Quebec statute
constitutionally inapplicable?

Decision:.
 Morguard requires more recognition and enforcement of judgments of
sister provinces due to the integrating character of our constitutional
arrangements. A province isn’t prohibited from enacting legislation that
has an effect on litigation in other provinces, but it must meet minimum
standards of order and fairness.

 The Quebec statute doesn’t meet order and fairness requirement


underlying comity.

 Discovery is a v. important tool in litigation, and the order and fairness


required by the common bonds that the constitution provides mean
Quebec’s blocking statute is not OK.

 If BC is an appropriate forum, then discovery is necessary as a part of


litigation, and its discovery rules reach to the rest of the country. The
constitutional considerations of Morguard trump the Quebec Act, so it is
not effective against another Canadian court that is the proper forum.

 This case confirms that there are constitutional limits on service ex juris.
This case and Morguard invent “constitutional inapplicability”.

Obiter: It may be the case that the federal government has the power to
legislate w/r/t enforcement of judgments nationally (476).

Muscutt v. Courcelles – Ont. CA (2002) – p. 484

Defines the real and substantial connection test.

The real and substantial connection test for jurisdiction is correlative to


the real and substantial connection test for enforcement of judgments
(like Braintech).

I: whether the Ont. courts should assume jurisdiction over out-of-province Ds


in claims for damage sustained in Ont. as a result of a tort committed
elsewhere.

This decision tries to give content to the R&S test while being mindful La
Forest’s statement that it may be impossible to define it with much certainty.

 Muscutt differentiates between Real and Substantial Connection test


and Forum Non-Conveniens:
o Real and Substantial Connection test is rule based, and must
be decided before looking at FNC considerations
o Forum Non-Conveniens is discretionary, and is only considered
once a court determines that it has jurisdiction

 SCC finds Ont. CPR 17.02(h) is procedural; service in accordance w.


rules of a court does not itself confer jurisdiction.

 VB discusses US minimal connection test: focused on miles. Our R&SC


test is focused on borders. Are we really concerned about bias etc. in
other forums? Should we only use a FNC test? (In US the concern about
home-court bias leads people to go to federal courts under their diversity
jurisdiction: when there are Ps and Ds from different states, either party
can opt to go to fed court, and you go to fed court in the state where the
action was started.) The question of whether or not we need both tests
underlies the decision here.

482-484: Good review of cases up to this point.

 Two main approaches to assumed jurisdiction:


o Personal Subjection: test focuses on connections between D
and forum. If there are none, substantial connection test fails
and D doesn’t have to litigate in the foreign jurisdiction even if
it’s much cheaper or more convenient for everyone else (this is
essentially the US approach, e.g. World Wide Volkswagen).
o Broader Approach: test that looks at all connections: between
the forum and D, P, witnesses etc. – a much lower standard: it
must be fair for the case to be heard in the province b/c it’s a
reasonable place of the action to take place.

Oakley v. Barry leading NS case on this: NS resident suffered


from sub-standard medical care given in NB. Held that NS was
proper place to hear the case, b/c traveling would be a hardship,
she would be suffering in NS, wouldn’t be that hard for NB doctors
to come defend in NS etc.

 Ont. C.A. adopts broader approach. There is a textual basis for this
approach. US approach based in constitutional provision that they
can’t lose property w/o due process of law; logic is that losing a suit in
another state would deprive one of due process of law. We don’t have
that, but we have all the constitutional principles from Morguard:
mobility rights, common market, common citizenship, etc.

 8-Part Real and Substantial Connection Test:


1. Connection between D and forum
2. Connection between P and forum
3. Unfairness to D if court assumes jurisdiction
4. Unfairness to P if court assumes jurisdiction
5. Connection of others to the forum
6. Court’s willingness to enforce a foreign judgment rendered
on the same basis. The standards are correlative, and a
court deciding whether or not to take jurisdiction should be
mindful of its own jurisprudence: if it has previously refused
to enforce judgment of another province in a similar case, it
shouldn’t expect
7. Whether case is interprovincial or international. R&SC test
between provinces is more lax: D who has to travel to
another province is still w/in same country, w. same
language, similar laws, procedure etc. A stronger connection
is necessary to find a R&S connection internationally.
8. Comity and standards of jurisdiction and enforcement
elsewhere: look at international standards. Here court looks
to Brussels Convention, and Draft of Hague Conference on
worldwide assumption of jurisdiction.

Smith v. Money Mart – Ont. CA (2006) – p. 507

 application of Muscutt factors


 finds a real and substantial connection for parent company that doesn’t have
operations in the province
o subsidiary company does and evidence that parent company has total
control
o fairness balancing favours the individual plaintiffs in Ont.
o the fact that it is international and not interprovincial favours a finding
that there is no real and substantial connection, but all other factors
favour the plaintiff

Spar Aerospace

on appeal from Quebec, where Spar sued US defendant in PQ court, saying


Quebec didn’t meet R&SC test. LeBel said:
1. R&SC test didn’t apply to Quebec’s Civil Code, not on constitutional basis
but rather because the provisions of the code take into account the factors
of the R&SC test. This is sketchy b/c you’re supposed to do test on case
by case basis, and it is impossible to say how the provisions will be
applied in the future.
2. R&SC test doesn’t apply outside of Canada. This would mean that foreign
Ds can’t rely on lack of R&S connection. LeBel doesn’t mention Muscutt
and accompanying cases, where there were a bunch of US defendants.
Other courts and parties to actions haven’t really picked up on this case,
which VB thinks is just plain wrong.

World-Wide Volkswagen v. Woodson – US SC (1980)

In US long-arm jurisdiction is limited by the “due process” clause in the 14th


amendment. No state can deprive any peson of property w/o due process of
law, which has been read to mean that nobody can have their property
(money, etc.) taken away by a court with which they have no connection.

- US takes personal subjection approach instead of broader Canadian


approach from Muscutt

Chapter 8

Evaluation of the rules we’ve learned so far:

A main difficulty of conflicts is that there is no universally accepted approach to


dealing w. the problems at hand. Start with some fundamental propositions:

Law is a rational enterprise


• Decisions should be made based on principles and reason
• Even though there are limits to rationality (limited information, equally
desirous results) we still want a structured way in which to decide things
• If things are decided rationally one has to respect the demands of the
judicial process – know what criteria one needs to make a reasoned
argument, what facts are necessary
• Also we can talk rationally about the solutions the law reaches for the
problems it has to solve

Based on the proposition that law is a rational enterprise, traditional conflicts


theory is unsatisfactory, e.g.
• Characterization:
o Fundamental to the operation of the rules, but nobody can
agree on how it should be carried out or even what it is
o We draw lines even when we’re not sure why we should draw
them: e.g. Block Brothers – does it really matter whether
something is substantive or procedural given that what the case
really needed to balance were the need to not allow unlicensed
real estate agents to practice in BC and the need to not allow
unjust enrichment.
• Renvoi:
o Indefensible: it’s only of limited application, but we’re not even
sure why
o Logically insoluble: how do you know when to stop? It could go
on forever

“The claim of this part of the materials is that traditional conflicts doctrine is
fundamentally incapable of faithfulness to the demands of the process of
adjudication and, hence, to the requirements of justice.

• The traditional theory’s claim that it leads to certainty of results is wrong


and illogical
• The American approach is good to look at b/c they started from a different
point than Canada did

Anda mungkin juga menyukai